It can be inferred that both authors would be most likely to agree with which one of the following statements regardi...

Alexandra on July 19, 2020

Explanation

Can this be explained please

Reply
Create a free account to read and take part in forum discussions.

Already have an account? log in

Shunhe on July 20, 2020

Hi @alliehall21,

Thanks for the question! So we’re looking for a statement that both authors would agree with regarding purple loosestrife. And we know that overall, on the big picture, they disagree about how dangerous its effects are. So we know that the similarity is going to have to lay somewhere else where they could both agree, some kind of incontrovertible evidence or something like that.

Now let’s take a look at (A). Would passages A and B agree with the statement that as purple loosestrife increases in North America, some wildlife populations tend to decrease? Well, first, let’s look at passage A. Clearly, passage A is going to agree with this statement. Look, for example, at lines 5-8, and lines 11-15. We’re told that purple loosestrife populations increasing has been associated with less native vegetation, reduced waterfowl and aquatic furbearer productivity, and degraded breeding habitats of several endangered species of vertebrates. So we know that passage A would agree with (A).

Now let’s see if passage B would agree with (A). Passage B is trying to say that the effects of purple loosestrife aren’t as bad as they seem. But even passage B can’t deny that there are some decreases in wildlife populations as purple loosestrife increases. Look, for example, at lines 53-55, where we’re told that “no bird species other than the canvasback has been identified in the literature as endangered by purple loosestrife.” Which means, of course, that the canvasback has been endangered by the purple loosestrife. And “some” on the LSAT just means at least one, so that’s enough for passage B to agree with (A). We’re also told in the following lines that it has an effect on furbearing animals in the next sentence. So both of the passages agree with (A), and so (A) is the correct answer.

Notice that (C) wouldn’t be agreed with by passage A, (B) and (D) wouldn’t be agreed with by passage B, and (E) isn’t mentioned at all by passage B, so each of these are wrong answers.

Hope this helps! Feel free to ask any other questions that you might have.